Difference between revisions of "2019 AMC 10B Problems/Problem 13"

m (Video Solution)
(14 intermediate revisions by 10 users not shown)
Line 1: Line 1:
Don't go breaking my heart
+
{{duplicate|[[2019 AMC 10B Problems|2019 AMC 10B #13]] and [[2019 AMC 12B Problems|2019 AMC 12B #7]]}}
 +
 
 +
==Problem==
 +
 
 +
What is the sum of all real numbers <math>x</math> for which the median of the numbers <math>4,6,8,17,</math> and <math>x</math> is equal to the mean of those five numbers?
 +
 
 +
<math>\textbf{(A) } -5 \qquad\textbf{(B) } 0 \qquad\textbf{(C) } 5 \qquad\textbf{(D) } \frac{15}{4} \qquad\textbf{(E) } \frac{35}{4}</math>
 +
 
 +
==Solution==
 +
 
 +
The mean is <math>\frac{4+6+8+17+x}{5}=\frac{35+x}{5}</math>.
 +
 
 +
There are three possibilities for the median: it is either <math>6</math>, <math>8</math>, or <math>x</math>.
 +
 
 +
Let's start with <math>6</math>.
 +
 
 +
<math>\frac{35+x}{5}=6</math> has solution <math>x=-5</math>, and the sequence is <math>-5, 4, 6, 8, 17</math>, which does have median <math>6</math>, so this is a valid solution.
 +
 
 +
Now let the median be <math>8</math>.
 +
 
 +
<math>\frac{35+x}{5}=8</math> gives <math>x=5</math>, so the sequence is <math>4, 5, 6, 8, 17</math>, which has median <math>6</math>, so this is not valid.
 +
 
 +
Finally we let the median be <math>x</math>.
 +
 
 +
<math>\frac{35+x}{5}=x \implies 35+x=5x \implies x=\frac{35}{4}=8.75</math>, and the sequence is <math>4, 6, 8, 8.75, 17</math>, which has median <math>8</math>. This case is therefore again not valid.
 +
 
 +
Hence the only possible value of <math>x</math> is <math>\boxed{\textbf{(A) }-5}.</math>
 +
 
 +
==Video Solution==
 +
https://youtu.be/mXvetCMMzpU?t=448
 +
 
 +
~IceMatrix
 +
 
 +
==See Also==
 +
 
 +
{{AMC10 box|year=2019|ab=B|num-b=12|num-a=14}}
 +
{{AMC12 box|year=2019|ab=B|num-b=6|num-a=8}}
 +
{{MAA Notice}}

Revision as of 22:37, 7 September 2020

The following problem is from both the 2019 AMC 10B #13 and 2019 AMC 12B #7, so both problems redirect to this page.

Problem

What is the sum of all real numbers $x$ for which the median of the numbers $4,6,8,17,$ and $x$ is equal to the mean of those five numbers?

$\textbf{(A) } -5 \qquad\textbf{(B) } 0 \qquad\textbf{(C) } 5 \qquad\textbf{(D) } \frac{15}{4} \qquad\textbf{(E) } \frac{35}{4}$

Solution

The mean is $\frac{4+6+8+17+x}{5}=\frac{35+x}{5}$.

There are three possibilities for the median: it is either $6$, $8$, or $x$.

Let's start with $6$.

$\frac{35+x}{5}=6$ has solution $x=-5$, and the sequence is $-5, 4, 6, 8, 17$, which does have median $6$, so this is a valid solution.

Now let the median be $8$.

$\frac{35+x}{5}=8$ gives $x=5$, so the sequence is $4, 5, 6, 8, 17$, which has median $6$, so this is not valid.

Finally we let the median be $x$.

$\frac{35+x}{5}=x \implies 35+x=5x \implies x=\frac{35}{4}=8.75$, and the sequence is $4, 6, 8, 8.75, 17$, which has median $8$. This case is therefore again not valid.

Hence the only possible value of $x$ is $\boxed{\textbf{(A) }-5}.$

Video Solution

https://youtu.be/mXvetCMMzpU?t=448

~IceMatrix

See Also

2019 AMC 10B (ProblemsAnswer KeyResources)
Preceded by
Problem 12
Followed by
Problem 14
1 2 3 4 5 6 7 8 9 10 11 12 13 14 15 16 17 18 19 20 21 22 23 24 25
All AMC 10 Problems and Solutions
2019 AMC 12B (ProblemsAnswer KeyResources)
Preceded by
Problem 6
Followed by
Problem 8
1 2 3 4 5 6 7 8 9 10 11 12 13 14 15 16 17 18 19 20 21 22 23 24 25
All AMC 12 Problems and Solutions

The problems on this page are copyrighted by the Mathematical Association of America's American Mathematics Competitions. AMC logo.png